????????????????????

Answers

Answer 1

we have

m=-2

point (1,2)

step 1

find the equation of the line in point slope form

y-3=-2(x-1)

convert to slope intercept form

y-3=-2x+2

y=-2x+2+3

y=-2x+5

To graph the line we need two points

Find the y-intercept (value of y when the value of x is equal to zero)

For x=0

y=5

so

the y-intercept is (0,5)

Find the x-intercept

Value of x when the value of y is equal to zero

For y=0

0=-2x+5

2x=5

x=2.5

the xintercept is (2.5,0)

Plot the points (0,5) and (2.5,0), join them and graph the line

using a graphing tool

see the attached figure

please wait a minute

????????????????????

Related Questions

What is the answer to this math problem which isn4/5+3/5=7/5=

Answers

Answer:

14/5 or, 2 4/5.

Step-by-step explanation:

4/5 + 3/5 = 7/5, 7/5 + 7/5 = 14/5

14/5 as a mixed number is 2 4/5.

I think you're talking about fractions so.

Write an equation for the line graphed below.

Answers

Answer:

y=-4/3x

Step-by-step explanation:

There is no b because it starts at the origin.

Answer:

-4/3x is the correct answer as provided by IceJadeKitsune

I am merely providing an explanation in case you are curious

Step-by-step explanation:

The equation for a line in slope-intercept form is

y = mx + b

where m is the slope and b the y-intercept

The slope of a line, m can be determined by what is called the rise/run ratio

The process is as follows

Take any two convenient points on the line and note their coordinatesLet's label the points as (x1, y1) and (x2, y2)The rise is the difference in the y values = y2 - y1 (The run is the difference in the x values = x2 - x1Divide this rise over run and you get the slope, mTo get b, find where the line crossed the y-axis and the value of b will be the value of y at that point.

The value of x will be 0 at the y-intercept

From the graph choose points origin (0, 0)  which is one point where the line crosses the y axis

From the graph we see that another distinct point where the line passes is at x = -3, y = 4 or the point(-3, 4)

Having got these two points we calculate
rise = 4 - 0 = 4
run = -3 - 0 = -3
Slope m = -4/3

Slope is negative because as y increases, x decreases

So the equation is y = -4/3x + b

Looking at the graph, we see that b = 0 since the graph passes through the origin as correctly stated by  IceJadeKitsune


Therefore the equation of the line is [tex]\boxed{\bold{y = -\dfrac{4}{3}x}}[/tex]

4. (01.02 LC)
Brandi earned $59.00 in interest after one and one half years on an account that paid 5.5% simple interest annually. Use the formula / -Prt to find Brand's principal balance. Round to the nearest hundredth. (1
point)
O $486.75
$535.23
$617.98
O $715.15

Answers

Simple interest is a clear and simple method for computing financial interest. Simple Interest is the amount returned for using the borrowed funds over a predetermined amount of time.

What is meant by simple interest?

Simple interest is, by definition, the amount of interest paid on a specific principal sum of money when an interest rate is applied. Compound interest, on the other hand, is the interest that is computed using both the principal and the interest that has accumulated over the preceding period.

Simple Interest is the amount returned for using the borrowed funds over a predetermined amount of time.

A = P(1 + rt)

Simple interest is a method for figuring out how much interest was paid on a sum of money during a specific time period at a specific rate. Simple interest has a constant principle amount. Simple interest is a clear and simple method for computing financial interest.

Therefore, the correct answer is option D. $715.15.

To learn more about Simple interest refer to:

https://brainly.com/question/25793394

#SPJ13

What does the dashers part of the figure represent

Answers

The figure represents : Line

What is a Line?

A line is an object in geometry that is indefinitely long and has neither breadth nor depth nor curvature. Since lines can exist in two, three, or higher dimensional environments, they are one-dimensional things. The term "line" can also be used to describe a line segment in daily life that contains two locations that serve as its ends.

from the figure we have to find that  whether its a line, line segment, vertex or ray

Line: A line is a perfectly straight, one-dimensional shape that extends infinitely in both directions and has no thickness. Sometimes a line is referred to as a straight line or, more formally, a right line.

Line segment: In other words, a line segment is a section or element of a line with two endpoints. A line segment, in contrast to a line, has a known length. A line segment's length can be calculated using either metric measurements like millimeters or centimeters or conventional measures like feet or inches.

Ray: A ray is a vector from a point to another point when seen as a vector. A ray is typically viewed in geometry as a half-infinite line, or half-line, with one of the two points and assumed to be at infinity.

Vertex: A vertex is a point on a polygon where two rays or line segments meet, the sides, or the edges of the object come together. Vertex is the plural form of vertices.

The figure represents a Line

Hence,  The dashers part of the figure represent a Line

To learn more about Line click on the link

https://brainly.com/question/2437195

#SPJ9

let f(x) =(4x"3+20)"2 and g(x) =4x"3+20.given that f(x)=(h°g)(x), find h(x)

Answers

[tex]h(x)=x^2[/tex]

Here, we want to find the function h(x)

From the question, we can see that the function f(x) is a composite function that was obtained by fitting g(x) into h(x)

What can we notice about g(x) and f(x)?

What we can see is that f(x) is the square of g(x)

Thus, what this mean is that h(x) = x^2

What are the six trigonometric ratios, and how are some of them related to each other(which are reciprocals of which)?

Answers

Explanation:

Consider the following right triangle:

In this triangle

x = adjacent side to the angle theta.

y = opposite side to the angle theta.

h= hypotenuse.

Now, by definition, we have the following trigonometric ratios:

[tex]\cos(\theta)=\frac{adjacent\text{ side to the angle }\theta}{hypotenuse}\text{ =}\frac{x}{h}[/tex][tex]\sin(\theta)=\frac{opposite\text{ side to the angle }\theta}{hypotenuse}=\frac{y}{h}[/tex][tex]tan(\theta)=\frac{opposite\text{ side to the angle }\theta}{adjacent\text{ side to the angle }\theta}=\text{ }\frac{y}{x}=\frac{y\text{ /h}}{x\text{ /h}}\text{ =}\frac{\sin(\theta)}{\cos(\theta)}[/tex]

and according to the above trigonometric ratio, we get:

[tex]cotan(\theta)=\frac{\cos(\theta)}{\sin(\theta)}[/tex]

On the other hand, we get the following reciprocals:

[tex]csc(\theta)=\frac{1}{\sin(\theta)}[/tex]

and

[tex]sec(\theta)=\frac{1}{cos(\theta)}[/tex]

we can conclude that the correct answer is:

Answer:

The six trigonometric ratios:

[tex]\cos(\theta)=\frac{adjacent\text{ side to the angle }\theta}{hypotenuse}\text{ }[/tex]

[tex]\sin(\theta)=\frac{opposite\text{ side to the angle }\theta}{hypotenuse}[/tex]

[tex]tan(\theta)=\frac{opposite\text{ side to the angle }\theta}{adjacent\text{ side to the angle }\theta}=\text{ }\frac{\sin(\theta)}{\cos(\theta)}[/tex]

[tex]csc(\theta)=\frac{1}{\sin(\theta)}[/tex]

[tex]sec(\theta)=\frac{1}{cos(\theta)}[/tex]

[tex]cotan(\theta)=\frac{\cos(\theta)}{\sin(\theta)}[/tex]

make a 2 column proofplease make it simple like JK is parallel to NM(given)

Answers

Explanation:

Since L is the midpoint of JM, then JL = LM.

Therefore,

Statement: JL = LM

Reason: L is the midpoint of JM

The lines JK and NM are parrallel; therefore, by the alternate interior angles theorem,

[tex]\angle LJK=\angle LMN[/tex]

Furthermore, since ∠JLK and ∠MLN are vertical angles,

[tex]∠JLK=∠MLN[/tex]

Now since ∠JLK = ∠MLN, ∠LJK = ∠LMN, and JL = LM, then by ASA postulate

[tex]\boxed{△JKL=△MNL.}[/tex]

Hence, our proof is complete!

What is a polygon with 10 sides called?dodecagonoctagontarragondecagon

Answers

A polygon with 10 sides is called a decagon.

Help me asp please!!

Answers

It is a function because it passes the vertical line test. The vertical line test says that if a vertical line is drawn on the graph anywhere, only one point of the graph will intersect with it.

The school record for the greatest number of jumping jacks in a row is 84 in 4 minutes. If the record for jumping jacks made is a constant ratio, how many jumping jacks did the record holder make in 1 minute?

Answers

The amount of jumping packs in 1 minute is 21

How to determine the amount of jumping packs in 1 minute?

From the question, the given parameters are:

Number of jumping packs in a row = 84 packs

Number of minutes = 4 minutes

The amount of jumping packs in 1 minute is the quotient of the number of jumping packs in a row and the number of minutes

This is represented as

Jumping packs in 1 minute = Number of jumping packs in a row /Number of minutes

Substitute the known values in the above equation

Jumping packs in 1 minute = 84/4

Evaluate

Jumping packs in 1 minute = 21

Hence, the jumping packs in 1 minute is 21

Read more about unit rates at

https://brainly.com/question/4895463

#SPJ1

The record holder makes 21 jumping jacks in 1 minute which is the ratio of the number of jumping jacks performed in a row to the total number of minutes.

What is the ratio?

A ratio is a relationship between two amounts that is represented by the division of one by the other.

The ratio of the total number of jumping jacks performed in a row to the total number of minutes determines how many jumping jacks should be performed in a minute.

No. of jumping packs in a row = 84 packs

No. of minutes = 4 minutes

So jumping packs in 1 minute = 84/4

Apply the division operation to get

Jumping packs in 1 minute = 21

Therefore, 21 jumping jacks should be performed in 1 minute.

Learn more about the Ratios here:

brainly.com/question/1504221

#SPJ1

PLEASE HELP!!!!!

Answer two questions about Equations AAA and BBB:

A. 4x+2=6-x
B. 5x+2=6

1) How can we get Equation BBB from Equation AAA?

Choose 1 answer:

(Choice A)
Add/subtract a quantity to/from only one side

(Choice B)
Add/subtract the same quantity to/from both sides

(Choice C)
Multiply/divide only one side by a non-zero constant

(Choice D)
Multiply/divide both sides by the same non-zero constant

Based on the previous answer, are the equations equivalent? In other words, do they have the same solution?

Choose 1 answer:

(Choice A)
Yes

(Choice B)
No

Answers

We can get equation B from equation A by Adding the same quantity to both sides. The equations are equivalent and have the same solution.

The equations are:

A : 4x+2 = 6-x

B : 5x+2 = 6

(1) We can get equation B from equation A by adding x to both sides of the equation.

We can verify this.

Add x to both sides of the equation A,

⇒ 4x+2 +x = 6-x+x

⇒ 5x+2 = 6

This is equation B.

So Add the same quantity to both sides of equation A to get equation B.

(2) We just added a same quantity on both sides of equation A. This will not change the equations. So equation A and equation B are equivalent. Also they have same solution.

Equation A: 4x+2=6-x

⇒ 4x+x = 6-2

⇒ 5x = 4

⇒ x = 4/5

Equation B: 5x+2 = 6

⇒ 5x = 6-2

⇒ 5x = 4

⇒ x = 4/5

So both have same solution.

Learn more about equivalent equation at https://brainly.com/question/2972832

#SPJ1

Donny the Dot Dude is saving for a llama. He puts$3,000 in a savings account that earns 12% simpleannual interest. How many years it will it take forhim to have the $4,440 he needs ifhe makes no additional depositsor withdrawals?A) 4 yearsSOB) 5 yearsC) 6 yearseboD) 2 yearsх

Answers

The simple interest formula is:

A = P(1 + rt)

where A is the final amount, P is the princiapal, r is the annual interest rate (as a decimal), and t is time in years.

Substituting with A = $4,440, P = $3,000, and r = 0.12 (= 12/100), we get:

[tex]\begin{gathered} 4440=3000\cdot(1+0.12\cdot t) \\ \frac{4440}{3000}=1+0.12\cdot t \\ 1.48-1=0.12\cdot t \\ \frac{0.48}{0.12}=t \\ 4\text{ years = t} \end{gathered}[/tex]

x + 3 = -7 Solve the inequality

Answers

The given inequality is expressed as

[tex]\begin{gathered} x\text{ + 3 }\leq\text{ - 7} \\ x\text{ }\leq\text{ - 7 - 3} \\ x\text{ }\leq\text{ - 10} \end{gathered}[/tex]

I think one is 10 I am just not sure though

Answers

Right Triangles

A right triangle is identified because it has an interior angle of 90° (right angle). The other two angles must be acute and add up to 90°.

In the special case that both acute angles have a measure of 45°, then we have an isosceles triangle, that is, both legs have the same length.

We can see one of the legs measures

[tex]L=5\sqrt[]{2}[/tex]

Being this an isosceles triangle, the other leg (the base of the triangle) also measures L, thus you should drag

[tex]5\sqrt[]{2}[/tex]

To the box at the bottom.

The other unknown length is the hypotenuse H. According to the Pythagorean's Theorem:

[tex]H^2=L^2+L^2=2L^2[/tex]

Substituting:

[tex]\begin{gathered} H^2=2(5\sqrt[]{2}^{})^2 \\ \text{Operating:} \\ H^2=2\cdot50=10 \\ \text{Thus:} \\ H=\sqrt[]{100} \\ H=10 \end{gathered}[/tex]

You should drag the length 10 to the hypotenuse.

How long ago, to the nearest year, was the artifact made?

Answers

Let's use the following formula:

[tex]A=A_0(0.5)^{\frac{t}{h}}[/tex]

where:

Ao= Initial amount

t = time

h = half-life

[tex]\begin{gathered} A=0.2A_0 \\ so\colon \\ 0.2A_0=A_0(0.5)^{\frac{t}{5730}} \end{gathered}[/tex]

solve for t:

[tex]\begin{gathered} 0.2=0.5^{\frac{t}{5730}} \\ \ln (0.2)=\frac{t}{5730}\ln (0.5) \\ t=5730\cdot\frac{\ln (0.2)}{\ln (0.5)} \\ t\approx13305 \end{gathered}[/tex]

A freshly brewed cup of coffee has temperature 95°C in a 20°C room. When its temperature is 67°C, it is cooling at a rate of 1°C per minute.

Let y = T − Ts, where T(t) is the temperature of the coffee in degrees Celsius at time t and Ts is the temperature of the surroundings in degrees Celsius. Find the values of A (in °C) and k for y(t) = Aekt.

Find: A and k

After how many minutes is the temperature of the coffee 67°C? (Round your answer to two decimal places.)
Answer:

Answers

The temperature of coffee will reach 67 °C

The rate of temperature change is most directly related to the difference between the body temperature and room temperature.

[tex]\frac{dT}{dt} =-k(T-T1)[/tex]

This equation's solution using the initial condition

T(0) = T0

T(t) = Tr + (T0 - Tr)

At the moment, the cooling rate

k × (T(t) - Tr)

We can write that according to the text

T1 = Tr + (T0 - Tr)

Solving the system of equations generates the unknown time:

t = [tex]\frac{69-20}{1} (ln\frac{95-20}{69-20})[/tex]

t = 21 min.

Hence the coffee will reach the temperature 67°C in 21 minutes.

Learn more about Thermodynamics at

https://brainly.com/question/1604031?referrer=searchResults

#SPJ1

ILL GIVE BRAINLIEST!!!!

Answers

Answer:

1 = yes

2 = no

3 = no

4 = yes

5 = yes

6 = no

7 = no

8 = no

Is 24x equivalent to 3(2x + 6x)?YesNo

Answers

Answer

Yes

Step-by-step explanation

Given the expression:

[tex]3(2x+6x)[/tex]

Combining similar terms:

[tex]3(8x)[/tex]

Multiplying:

[tex](3\cdot8)x=24x[/tex]

A poster is 3 feet wide and 12 feet long. What are the dimensions if the poster is enlarged by a factor of 5/2?

Answers

To be able to get the enlarged dimensions, we will multiply the dimension of the poster by the given scale factor 5/2.

We get,

[tex]\text{Width = 3 ft. x }\frac{5}{2}\text{ = }\frac{15}{2}\text{ ft. or 7.5 ft.}[/tex][tex]\text{Length = 12 ft. x }\frac{5}{2}\text{ = }\frac{60}{2}\text{ ft. or 30 ft.}[/tex]

Therefore, the enlarged dimension of the poster will be 7.5 ft. x 30 ft.

find the height of a square pyramid with V of 60 cm3 and base side length S of 6cm

Answers

Using Volume formula, the height of the square pyramid is 5 cm.

What is volume of a square pyramid?

If the side of the square base of the pyramid is b and height of the pyramid be h

Volume of the square pyramid , V is given by the below formula.

V = 1/3(b²h)

Given

Volume of the square pyramid, V = 60 cm³

base side is given , b = 6 cm

let Height of the square pyramid be h

⇒ V =  1/3(b²h)

⇒ 3V = b²h

⇒3V/b² = h

⇒ 3(60)/ 6² = h

⇒ h = 180/36 = 5

Therefore, the height of the given square pyramid id 5 cm.

Also, learn more about Volume of Pyramid from the link below:

https://brainly.com/question/17615619

#SPJ1

Select the correct answer. Which graph represents the solutions to this equation? x2 + 8x = -20 A. Linear-quadratic system graph shows upward parabola with vertex at (minus 2, 4) and passing through x and y-axis (minus 8, 0), and (0, 0) B. Linear-quadratic system graph shows upward parabola with vertex at (minus 4, 4) and passing through (minus 2, 8), and (minus 6, 8) C. Linear-quadratic system graph shows upward parabola with vertex at (4, 0) and passing through (1, 4), and (6, 4) D. Linear-quadratic system graph shows a downward parabola with vertex at (0, 8) which intercepts the x-axis at 3 and minus 3 units. Reset Next

Answers

A graph which represents the solutions to this quadratic equation (x² + 8x = -20) is: B. Linear-quadratic system graph shows upward parabola with vertex at (minus 4, 4) and passing through (minus 2, 8), and (minus 6, 8).

What is a graph?

A graph can be defined as a type of chart that's commonly used to graphically represent data on both the horizontal and vertical lines of a cartesian coordinate, which are the x-axis and y-axis.

In Mathematics, the graph of any quadratic function or equation always forms a parabola because it is a u-shaped curve.

In conclusion, we can reasonably infer and logically deduce that the given quadratic equation is modeled by an upward parabola with its vertex at (-4, 4) as shown in the graph attached below.

Read more on graphs here: brainly.com/question/4546414

#SPJ1

someone help please

Answers

Answer:

Angle 3 = 95° (vertically opposite angles)

Angle 2 and 4 = 180 - 95

= 85° (adjacent angles on straight line)

Angle 5 = 180 - 144

= 36° (adjacent angles on straight line)

Angle 6 = 180 - angle 3 - angle 5

= 180 - 95 - 36

= 49° (sum of angles in triangle=180)

Angle 1 = 180 - 90 - angle 6

= 180 - 90 - 49

= 41° (sum of angles in triangle=180)

Angle 7 = 180 - 38 - angle 5

= 180 - 38 - 36

= 106° (sum of angles in triangle=180)

How can you use the Power of a Quotient, Quotient of Powers, Zero Exponent Laws Identity Exponent and to evaluate numerical expressions with whole-number exponents?​

Answers

Exponents and powers are terms that occasionally get used interchangeably, which can be confusing.

Mathematics uses expressions called powers, where n is the exponent and x is the base. When a number or variable is multiplied repeatedly, it is referred to as a power. The exponent of power tells us how many times to multiply the base by itself.

You can interpret the term as "x to the power." The exponent (n) is written smaller and at the head of the line using superscript, whereas the base (x) is printed in full size (if you are typing it on a computer). For instance, it is written as x squared or x to the second power, which in reality means that the value of x is multiplied by an amount equal to the exponent's value.

If the base is a number: In this situation, all you have to do to discover the solution is multiply the base by itself as many times as the exponent's value.

If the base is a variable, you must first replace the variable with a value before continuing.

Learn more about powers and exponents at

https://brainly.com/question/15722035?referrer=searchResults

#SPJ1

7. Solve for value of tan A 8. Solve for Cos Q

Answers

7. Tan A = opposite/adjecent

Therefore Tan A = 24/7

• Option number (2) is correct .

8. Cos Q = adjacent/hypotaneus

where hypotaneus side : r^2 = x^2+y^2

r^2 = 12^2 +5^2= 169

r =√169 = 13

Now Cos Q = 12/13

• Option 4 is correct .

I believe you start with 2.4 million has initial start value? Not sure cause it worded weirdly and English not very good

Answers

2,466,000

Here, we want to know what is to be used as the initial value

The general exponential equation should be in the form;

[tex]P=I(1+R)^n[/tex]

where P is the population at a certain year

I is the initial population which is 2,466,000 in this case

R is the rate of increase

n is the number of years

which transformations had to occur for the blue triangle to become the purple triangle? [note: all rotations are about origin]

Answers

Let:

A = (-4,3)

B = (-1,3)

C = (-1,1)

after a 90 clockwise rotation:

A = (-4,3)--------->(y,-x)------->(3,4)

B = (-1,3)----------->(y,-x)------>(3,1)

C = (-1,1)------------>(y,-x)------>(1,1)

After a translation 3 units down and 2 units left:

(3,4)------>(x-2,y-3)------->(1,1)

(3,1)------>(x-2,y-3)------->(1,-2)

(1,1)------>(x-2,y-3)------->(-1,-2)

---------------------------

Therefore, the answer is D

v=LMH for L

thankssssss .

Answers

L= MH/ v Explanation : you move the v to divide it ( the reciprocal ) and then you’ll move the L to where the v was to replace it and that’s how you solve for L

Middle School Debate Club 30% members are in 6th grade. If there are 12 6th graders in The Debate Club, how many total members are there ?

Answers

We know that 30% of the members are in 6th grade, and add up to 12 members.

Lets X be the total members.

Then 0.3*X are in 6th grade, and this is equivalent to 12 members.

NOTE: 0.3 is the decimal form of 30%.

We can write:

[tex]\begin{gathered} 0.3\cdot X=12 \\ X=\frac{12}{0.3} \\ X=40 \end{gathered}[/tex]

Answer: there are 40 members in the Debate Club.

I got -52.7 degrees but I want to make sure. thank you

Answers

The given vector is

[tex]m=\langle16,-21\rangle[/tex]

The formula to find the direction angle is

[tex]\theta=\tan ^{-1}|\frac{y}{x}|[/tex]

Replacing each coordinate, we have

[tex]\theta=\tan ^{-1}|\frac{-21}{16}|=52.7[/tex]Therefore, the direction angle is 52.7°, approximately.

Find the present value given the following:Amount needed: $9,350Time in years: 3Interest: 5%Compounded: semiannually

Answers

Solution:

Amount needed (P): $9,350

Time in years (n): 3

Interest (r): 5%

Compounded: semiannually

To find the Amount (A).

we have the formula A, we get,

[tex]A=P(1+\frac{\frac{r}{2}}{100})^{2n}[/tex][tex]=9350(1+\frac{2.5}{100})^{2(3)}[/tex][tex]=9350(\frac{102.5}{100})^6[/tex][tex]=9350(1.025)^6[/tex][tex]=9350(1.15969)[/tex][tex]A=10,843.13[/tex]

The present value is $10,843.133

Other Questions
Evaluate 14xy if x=23 and y=35 . Write your answer as a fraction in simplest form. PLEASE HURRY THIS IS AN IMPORTANT TEST: Click on the conclusion that can be drawn about why the author thought thatch roofs were used on roundhouses please look at screenshots Using this phospholipid bilayer cross-section diagram as a reference, which letter best represents an integral protein that spans the membrane?ABCDEFG If triangle LMN has an obtuse angle at vertex M, which statements could be true? Check all that apply. Select the correct figures how did women participate in protesting the townshend acts? refusing to marry or have children acting as representatives in colonial legislatures spinning homespun clothing to aid the trade boycott they did not participate Phrases and clauses Coulomb's law can be used to calculate the _____________. work out the sale has been decreased 20% of 120 [tex]-19.35+16.04[/tex] Describe in extension each of the following sets:a) A is the set of positive integers less than 8.C) C is the set of digits of the number 12353888D) D is the set of courses you are enrolled in. Givin f(x) = 1/x+5 and g(x)=x-2. What are the restrictions of the domain of f(g(x))?OX -5OX -3OX 2OThere are no restrictions solve the triangle. round to the nearest tenth A B 16 14 A roller coaster car is traveling through a loop at 13 m/s. If the loop has a 23m radius, what centripetal force will the 53kg rider feel? A circle has a center at (4, -7) and a radius of 4 units. What is the equation of this circle?O (4) + (y + 7) = 16O (x + 4) + (y + 7) = 4O(x-4) + (y - 7) = 16O(x + 4) + (y - 7) = 4 For this exponential function,what is the output value (y),when the input value (x) is 3?y = 5.2x(3, [?])Enter Which of the following best describes AD?17D17A. MedianB. AltitudeC. Perpendicular bisectorD. Angle bisector In the expression below, what operation should be done first? 4 + 8 2 drag the words or phrases on the left to the appropriate blanks on the right to complete the sentences. words or phrases can be used once, more than once, or not at all.